Matemática, perguntado por carlaferraz0101, 4 meses atrás

gente ajuda pf!!

PUCPR ​

Anexos:

Soluções para a tarefa

Respondido por Vulpliks
1

Primeiro tenho de dar os créditos ao pessoal do forumeiros, eles me deram a luz para entender essa questão.

É um problema de função composta. Contudo, a função composta resultante vai depender da paridade de n.

Vamos primeiro investigar a primeira função, imagine que n seja ímpar:

f(n) = n + 3 \text{, n impar}

Mas se n é ímpar, a soma de um ímpar com outro ímpar só pode resultar em um número par. Por exemplo: 1 + 3 = 4 (par), 3 + 3 = 6 (par), 5 + 3 = 8 (par).

Assim sendo, a primeira composta, f(f(n)) vai ser:

f(f(n)) = \dfrac{f(n)}{2} = \dfrac{n+3}{2}

Mas agora, como n é ímpar, e n + 3 é par, temos que o resultado dessa função pode ser par ou ímpar:

a) Se n = 1, 5, 9, 13,..., então \dfrac{n+3}{2} é par.

b) Se n = 3, 7, 11, ..., então \dfrac{n+3}{2} é ímpar.

Assim sendo, no primeiro caso (resultado par), usaremos a função de baixo:

a) f(f(f(n))) = \dfrac{f(f(n))}{2} = \dfrac{\dfrac{n+3}{2}}{2} = \dfrac{n+3}{4}

 f(f(f(n))) = n

 \dfrac{n+3}{4} = n

n + 3 = 4 \cdot n

4 \cdot n - n = 3

3 \cdot n = 3

 n = \dfrac{3}{3}

\boxed{n_1 = 1}

No segundo caso (resultado ímpar), usaremos a função de cima:

b)f(f(f(n))) = f(f(n)) + 3 = \dfrac{n+3}{2} + 3

 f(f(f(n))) = n

\dfrac{n+3}{2} + 3 = n

\dfrac{n+3}{2} = n - 3

n + 3 = 2 \cdot (n - 3)

n + 3 = 2 \cdot n - 6

2 \cdot n - n = 6 + 3

\boxed{n_2 = 9}

Ok, mas note que 9 é um número pertencente ao caso a) e não ao caso b). Então as chances disso ocorrer são nulas, e essa solução é descartada!

Ok, ok, mas isso foi apenas considerando que o n inicialmente era ímpar. Mas e agora, se for par? Nesse caso, usaremos a função de baixo:

f(n) = \dfrac{n}{2}\text{, n par}

Mas assim:

c) se n = 2, 6, 10, 14, 18..., o resultado disso será ímpar.

d) Caso n = 0, 4, 8, 12, 16, o resultado será par.

Então, para c):

f(f(n)) = \dfrac{n}{2} + 3

E como o resultado de \dfrac{n}{2} é ímpar, a soma de dois números ímpares é par. Por conseguinte:

f(f(f(n))) = \dfrac{\dfrac{n}{2} + 3}{2}

Assim sendo:

f(f(f(n))) = n

\dfrac{\dfrac{n}{2} + 3}{2} = n

\dfrac{n}{2} + 3 = 2 \cdot n

2 \cdot n - \dfrac{n}{2} = 3

\dfrac{3 \cdot n}{2} = 3

\boxed{n_3 = 2}

Finalmente, para o caso d):

f(f(n)) = \dfrac{\dfrac{n}{2}}{2} = \dfrac{n}{4}

E como nessa condição, n = 4, 8, 12, 16, 20..., o resultado pode ser tanto par quanto ímpar:

I) Se n = 4, 12, 20..., o resultado é ímpar, e:

f(f(f(n))) = \dfrac{n}{4} + 3

f(f(f(n))) = n

\dfrac{n}{4} + 3 = n

n - \dfrac{n}{4} = 3

\dfrac{3 \cdot n}{4} = 3

\boxed{n_4 = 4}

II) Se n = 8, 16, 24,... o resultado é par, e:

f(f(f(n))) = \dfrac{\dfrac{n}{4}}{2} = \dfrac{n}{8}

f(f(f(n))) = n

\dfrac{n}{8} = n

Isso só suporta uma única solução:

\boxed{n_5 = 0}

Então no final chegamos a 5 soluções, mas n_2 foi descartada. Logo, apenas 4 soluções são válidas!

Alternativa B


carlaferraz0101: muito obrigada mais uma vez!!!! você salvou meus estudos em matemática
Perguntas interessantes